Đến nội dung

truongphat266

truongphat266

Đăng ký: 06-01-2023
Offline Đăng nhập: Hôm qua, 14:50
***--

#743964 Ước nguyên tố của $a_i + a_j$ lớn hơn $l$

Gửi bởi truongphat266 trong 04-03-2024 - 22:26

Cho $a_1<a_2<\cdots$ là dãy các số nguyên dương tăng vô hạn. Chứng minh rằng với mọi số nguyên dương $l$ thì luôn tồn tại $i \neq j$ với $1 \leq i,j $ thỏa mãn $a_i + a_j$ có một ước nguyên tố lớn hơn $l$. 




#741418 $(\frac{a+b}{a+c}+\frac{a+c}...

Gửi bởi truongphat266 trong 16-09-2023 - 23:13

Cho $a,b,c$ thực dương thỏa mãn$(a+b)(b+c)(c+a)=8$. CMR 

$(\frac{a+b}{a+c}+\frac{a+c}{a+b}-1)(\frac{b+c}{b+a}+\frac{b+a}{b+c}-1)(\frac{c+b}{c+a}+\frac{c+a}{c+b}-1)\geq \frac{8abc}{(a^2+bc)(b^2+ac)(c^2+ab)}$

Ta có bổ đề sau: Cho $a,b,c \geq 0$ thì có: $$\frac{1}{(c+a)^2}+\frac{1}{(b+a)^2}\geq \frac{1}{a^2+bc}\Leftrightarrow \frac{(a^2-bc)^2+bc(b-c)^2}{(a+b)^2(a+c)^2(a^2+bc)}\geq 0.$$

Áp dụng bổ đề:

$$VT=\prod _{cyc}(a+b)(a+c)(\frac{1}{(c+a)^2}+\frac{1}{(b+a)^2}-\frac{1}{(c+a)(b+c)})\geq \prod_{cyc} \frac{a(b+c)}{a^2+bc}=VP.$$

Có điều phải chứng minh!




#741293 Vẻ đẹp bất đẳng thức Nesbitt - Kí ức THCS

Gửi bởi truongphat266 trong 03-09-2023 - 22:14

bài 1

chuẩn hóa a+b+c=3, tồn tại t: $ab+bc+ca=3-3t^2$ với $0\le t<1$

bdt cần chứng minh $\Leftrightarrow \frac{36-9t^2}{9(1-t^2)-abc}+2(1-t^2) \ge \frac{13}{2}$

theo bổ đề chặn tích thì $abc\ge (1-2t)(1+t)^2\Rightarrow \frac{36-9t^2}{9(1-t^2)-abc}+2(1-t^2)\ge \frac{36-9t^2}{9(1-t^2)-(1-2t)(1+t)^2}+2(1-t^2)$ 

chứng minh $\frac{36-9t^2}{9(1-t^2)-(1-2t)(1+t)^2}+2(1-t^2) \ge \frac{13}{2} \Leftrightarrow \frac{(2t-1)^2t^2}{2(2-t)(1+t)}\ge 0$

 

từ bài 1 suy ra bài 4 theo bdt Holder

Lời giải thật sự rất hay. Em xin phép được sửa lại đề câu 2 và câu 3 ạ, do trong quá trình đổi đề em ẩu nên sai quá chừng. Xin lỗi mọi người nhiều ạ!

Em xin được phép giải bài 2 và 3:

 

Bài 2:

 

Có:$$VT \geq \frac{(a+b+c)^2}{2(ab+bc+ca)}=\frac{3}{2}+\frac{(a+b+c)^2-3(ab+bc+ca)}{ab+bc+ca+(ab+bc+ca)}$$

$$\geq \frac{3}{2} +\frac{a^2+b^2+c^2-ab-bc-ca}{a^2+b^2+c^2+ab+bc+ca}=VP.$$

 

Bài 3: (Nguyen Viet Hung)

 

Có: $$VT\geq \frac{(a+b+c)^2}{2(ab+bc+ca)}+\frac{ab+bc+ca}{2(a^2+b^2+c^2)}$$

Cần chứng minh: $$\frac{(a+b+c)^2}{2(ab+bc+ca)}+\frac{ab+bc+ca}{2(a^2+b^2+c^2)}\geq 2\Leftrightarrow (a^2+b^2+c^2-ab-bc-ca)^2 \geq 0.$$

 

Một số bài tiếp theo:

 

Bài 4: $$\frac{a}{b+c}+\frac{b}{a+c}+\frac{c}{a+b}+\frac{3\sqrt[3]{a^2b^2c^2}}{2(a^2+b^2+c^2)} \geq2 $$ (uvw)

Bài 5: $$\frac{a}{b+c}+\frac{b}{c+a}+\frac{c}{a+b}\geq \frac12+\frac{a^3+b^3+c^3+3abc}{ab(a+b)+bc(b+c)+ca(c+a)}$$

Bài 6: $$\frac{a}{b+c}+\frac{b}{c+a}+\frac{c}{a+b} \geq \frac{bc}{a^2+bc}+\frac{ca}{b^2+ca}+\frac{ab}{c^2+ab}$$

Bài 7:(FuadAnzurov2003 - AoPS) $$\frac{a}{b+c}+\frac{b}{c+a}+\frac{c}{a+b} \ge \frac{3}{2}+\frac{9(b-c)^2}{16(a+b+c)^2}+\frac{9(c-a)^2}{16(a+b+c)^2}+\frac{9(b-c)^2}{16(a+b+c)^2}$$

Em xin cám ơn ạ!!




#741283 Vẻ đẹp bất đẳng thức Nesbitt - Kí ức THCS

Gửi bởi truongphat266 trong 02-09-2023 - 20:20

Chào mọi người mình là Lục Trường Phát, học sinh THPT chuyên Lê Hồng Phong (TPHCM).Hôm nay mình thấy diễn đàn dạo này nóng ít quá, nên mình góp một bài cho diễn đàn vui vậy, bài viết về một bất đẳng thức quen thuộc lúc THCS nhưng sẽ mạnh hơn nhé và nhìn lại những gì ta đã đi qua, một thời đáng nhớ!!!

 

Bài toán quen thuộc: Cho $a,b,c \geq 0.$ Chứng minh rằng: $\frac{a}{b+c}+\frac{b}{c+a}+\frac{c}{a+b}\geq \frac{3}{2}.$

Đây là bất đẳng thức Nesbitt vô cùng quen thuộc và được sử dụng rất nhiều cùng với vô số các cách chứng minh khác nhau. Chúng ta hãy cùng xem qua nhé:

(Mọi $a,b,c$ trong đây đều không âm)

Điều kiện thú vị: (kết quả của sqing - AoPS)

Điều kiện như trên nhưng chỉ thêm một thứ nữa là $a \geq b+c.$ thì có 2 kết quả sau:

 $$\frac{a}{b+c}+\frac{b}{c+a}+\frac{c}{a+b}\geq \frac{5}{3}$$

$$\frac{a}{b+c}+\frac{2b}{c+a}+\frac{2c}{a+b}\geq \frac{7}{3}$$

Và đây là chứng minh: 

Kết quả thứ nhất: Do vai trò của $b,c$ như nhau nên ta "mạnh dạn" Cauchy - Schwarz cho 2 lượng có tử $b,c$ được: $$VT\geq \frac{a}{b+c}+\frac{(b+c)^2}{a(b+c)+2bc}\geq\frac{a}{b+c}+\frac{(b+c)^2}{a(b+c)+2(\frac{b+c}{2})^2}=\frac{a}{b+c}+\frac{1}{\frac{a}{b+c}+\frac{1}{2}}$$

Tới đây là bước AM - GM quen thuộc trong dạng điểm rơi:

$$=\frac{5}{9}(\frac{a}{b+c}+\frac{1}{2})+[\frac{4}{9}(\frac{a}{b+c}+\frac{1}{2})+\frac{1}{\frac{a}{b+c}+\frac{1}{2}}]-\frac{1}{2}\geq \frac{5}{3}.$$

 

Kết quả thứ hai: chứng minh tương tự kết quả 1!! (Bạn đọc tự chứng minh)

 

Ngoài ra tạp chí Crux cũng đề cập đến một dạng bất đẳng thức quen thuộc này, trong sách "Sáng tạo bất đẳng thức" của Phạm Kim Hùng cũng đề cập đến:

Đề bài chỉ cho thêm $x,y,z \geq 0$. Tìm giá trị nhỏ nhất của: $$\frac{xa}{b+c}+\frac{yb}{c+a}+\frac{zc}{a+b}(*)$$

 

Đây là một dạng toán có thể đem đi chế đề mà không ai hay biết:) nhưng bài này thực sự nếu thay $x,y,z$ thành các biến $f(a;b;c)$ thì sẽ rất khó!!

 

Lời giải ta vẫn ưu tiên sử dụng Cauchy - Schwarz bằng việc thêm bớt để có nhân tử:

 

Có: $$(*)=(\frac{xa}{b+c}+x)+(\frac{yb}{c+a}+y)+(\frac{zc}{a+b}+z)-(x+y+z)$$

$$=\frac{1}{2}[(b+c)+(c+a)+(a+b)](\frac{x}{b+c}+\frac{y}{c+a}+\frac{z}{a+b})-(x+y+z)\geq \frac{(\sqrt{x}+\sqrt{y}+\sqrt{z})^2}{2}-(x+y+z).$$

 

Dấu bằng xảy ra khi:$\frac{b+c}{\sqrt{x}}=\frac{c+a}{\sqrt{y}}=\frac{a+b}{\sqrt{z}}.$

 

Qua đây có thể thấy Nesbitt được biến tấu thành những dạng vô cùng độc đáo, ngoài ra ta có kết quả thú vị nữa:

Lúc này ta cho thêm điều kiện: $a+b+c=3$

Thì với mọi $k$ không âm có: $$\frac{a^{k+1}}{b^{k}+c^{k}}+\frac{b^{k+1}}{c^{k}+a^{k}}+\frac{c^{k+1}}{a^{k}+b^{k}}\geq \frac{3}{2} (**)$$

Lời giải bài này vô cùng đơn giản: Chứng minh theo tư tưởng của Nesbitt, ta lấy vế trái trừ vế phải được:$$\sum _{cyc}(\frac{a^{k+1}}{b^k+c^k}-\frac{1}{2})=\sum_{cyc}(\frac{(a^k-b^k)(a^{k+1}-b^{k+1}+c^k(a-b))}{(b^k+c^k)(c^k+a^k)})=\sum_{cyc}(a-b)^2.S_a\geq 0.$$

Một lời chứng minh vô cùng đơn giản, bài này ta còn có thể giải quyết bằng bất đẳng thức hoán vị hoặc Chebyshev (Bạn đọc tự chứng minh)

 

Một kết quả khá thú vị mà chắc hẳn ai cũng từng làm qua. Với $n \geq 2$ thì có:

$$\sqrt[n]{\frac{a}{b+c}}+\sqrt[n]{\frac{b}{c+a}}+\sqrt[n]{\frac{c}{a+b}}\geq 2.$$

Ta giả sử: $a \geq b \geq c$ thì có: $a^{n-1}(b+c) , b^{n-1}(c+a) \geq c^{n-1}(a+b)$

Theo bất đẳng thức Bernoulli thì có: $(1+\frac{x}{n})^n \geq 1+x$ nên suy ra được $(1+x)^{\frac{1}{n}} \leq 1+ \frac{x}{n}$

Hay từ đây ta có được: $$(\frac{c+a}{b+c})^{\frac{1}{n}}\leq (\frac{a}{b+c})^{\frac{1}{n}}+\frac{1}{n}(\frac{c^n}{a^{n-1}(b+c)})^{\frac{1}{n}}\leq(\frac{a}{b+c})^{\frac{1}{n}}+\frac{1}{2}(\frac{c}{a+b})^{\frac{1}{n}}.$$

Tương tự cũng có được: $$(\frac{b+c}{c+a})^{\frac{1}{n}}\leq (\frac{b}{c+a})^{\frac{1}{n}}+\frac{1}{2}(\frac{c}{a+b})^{\frac{1}{n}}.$$

Cộng các bất đẳng thức trên lại, kết hợp AM - GM 2 số có điều phải chứng minh.

 

Một bài nữa cũng là một vẻ đẹp của Nesbitt nhưng được làm theo cách khác nữa, ta thêm một lượng ngược dấu:

$$\frac{a}{b+c}+\frac{b}{c+a}+\frac{c}{a+b}+\sqrt{\frac{ab+bc+ca}{a^2+b^2+c^2}} \geq \frac{5}{2}.$$

Bằng việc nâng bậc cụm Nesbitt và Cauchy - Schwarz kết hợp một chút cân bằng điểm rơi ta có điều phải chứng minh!!

Ta thử đổi lượng khác: $$\frac{a}{b+c}+\frac{b}{c+a}+\frac{c}{a+b}+\frac{9abc}{2(a+b+c)(ab+bc+ca)}\geq 2.$$

Bằng phương pháp đổi biến p,q,r thì ta thu lại được bất đẳng thức Schur. Ngoài ra còn có cách bằng SOS (Bạn đọc tự chứng minh)

 

Một bất đẳng thức được làm chặt lên, ta thêm lượng bình phương bên phải:

$$\frac{a}{b+c}+\frac{b}{c+a}+\frac{c}{a+b}\geq \frac{3}{2}+\frac{(b-c)^2}{2(b+c)^2}$$

Điều này tương đương:$$\frac{b(a-b)^2}{(c+a)(b+c)^2}+\frac{c(c-a)^2}{(a+b)(b+c)^2} \geq 0.$$

 

Sau đây là một sự pha trộn giữa Nesbitt và Schur: Ta thêm điều kiện: $ab+bc+ca\neq 0.$ Thì có:$$\frac{a}{b+c}+\frac{b}{c+a}+\frac{c}{a+b} \geq \frac{2(a+b+c)}{a+b+c+\sqrt[3]{abc}}.$$

Bạn chỉ cần Cauchy - Schwarz rồi Schur kết hợp thêm AM - GM là xong.

 

Một bất đẳng thức thú vị phù hợp cho lớp 8: Xét trường hợp: $a>b>c$ thì có:

$$\frac{a}{b-c}+\frac{b}{a-c}+\frac{c}{a-b}>2.$$

Bạn cũng chỉ cần Cauchy - Schwarz rồi nhân chéo thôi!

 

Để làm bất đẳng thức này tổng quát hơn, ta xét trường hợp $a,b,c$ là các số thực phân biệt thì có: $$|\frac{a}{b-c}|+|\frac{b}{c-a}|+|\frac{c}{a-b}| \geq 2.$$

Ta đặt:$(\frac{a}{b-c};\frac{b}{c-a};\frac{c}{a-b}) \mapsto (x;y;z)$

Vậy có: $$VT^2= (x+y+z)^2+2+2\sum_{cyc}|xy|\geq 4$$ hay ta có điều phải chứng minh.

Hãy thử trong trường hợp 4 số.

 

Một bất đẳng thức Nesbitt dưới góc nhìn của Muirhead:

$$\frac{b+c}{a}+\frac{c+a}{b}+\frac{a+b}{c} \geq \frac{a}{b+c}+\frac{b}{c+a}+\frac{c}{a+b}+\frac{9}{2}.$$

Chỉ cần quy đồng hết lên thì bạn sẽ nhận lại bất đẳng thức Muirhead!!! (Bạn đọc tự chứng minh)

 

Quay lại bài toán này:$$\sqrt[n]{\frac{a}{b+c}}+\sqrt[n]{\frac{b}{c+a}}+\sqrt[n]{\frac{c}{a+b}}\geq 2.$$

Ta làm mạnh thêm chút nữa bằng việc thêm điều kiện: $ab+bc+ca=1$ thì ta thu được: $$\sqrt{\frac{a}{b+c-2abc}}+\sqrt{\frac{b}{c+a-2abc}} +\sqrt{\frac{c}{a+b-2abc}} \geq 2$$

Bài này chỉ việc sử dụng đồng bậc, Holder và cuối cùng là Muirhead (Bạn đọc tự chứng minh) 

 

Một bài toán thú vị khác của 2012: Ta thêm điều kiện $abc=1$ thì thu được:

$$\frac{a^5b^5}{a^2+b^2}+\frac{b^5c^5}{b^2+c^2}+\frac{c^5b^5}{c^2+a^2}\geq \frac{3}{2}.$$

Bằng việc đổi biến:$(a;b;c) \mapsto (\frac{1}{x};\frac{1}{y};\frac{1}{z})$ kết hợp với 2 lần Chebyshev thì ta có điều phải chứng minh.

 

Ta có một bài toán khác: 

$$\frac{a}{b+c}+\frac{b}{c+a}+\frac{c}{a+b}+\frac{4abc}{(a+b)(b+c)(c+a)}\geq 2.$$

Bằng việc quy đồng ta thu được Schur.

 

Ta đổi lượng: $\frac{4abc}{(a+b)(b+c)(c+a)} \mapsto \frac{3\sqrt[3]{abc}}{2(a+b+c)}$ thì ta vẫn thu được kết quả trên vì: $\frac{3\sqrt[3]{abc}}{2(a+b+c)} \geq \frac{4abc}{(a+b)(b+c)(c+a)}$

 

Đây là một kết quả hay tiếp theo: $$\frac{a}{b+c}+\frac{b}{c+a}+\frac{c}{a+b}\leq \frac{1}{2}(\frac{a^2}{bc}+\frac{b^2}{ca}+\frac{c^2}{ab}).$$

Ta sử dụng Cauchy - Schwarz trực tiếp cho từng cặp bên vế phải thì thu được các lượng đối xứng, tiếp túc Cauchy - Schwarz trực tiếp cho từng cặp đối xứng chung tử vừa mới tạo được thì có điều phải chứng minh!

 

Một kết quả cực kì đẹp trong tam giác: Ta đổi điều kiện $a,b,c$ thành 3 cạnh tam giác thì với mọi 
$n$ nguyên dương thì có: $$(\frac{a}{b+c})^n+(\frac{b}{c+a})^n+(\frac{c}{a+b})^n <2.$$

Cách chứng minh bằng việc ta đổi biến: $(\frac{a}{b+c};\frac{b}{c+a};\frac{c}{a+b}) \mapsto (x;y;z)$ Điều kiện: $x,y,z \in (0;1)$ 

Không mất tính tổng quát giả sử: $x \geq y \geq z$

Từ đây có giả thiết mới: $\frac{1}{x+1}+\frac{1}{y+1}+\frac{1}{z+1}=2.$

Ta đi chứng minh 2 điều sau: $x+y+z <2$ và $x^{n+1} + y^{n+1}+z^{n+1} < x^n+y^n+z^n$

Bất đẳng thức đầu tiên, ta lấy vế phải trừ vế trái:

Thì có: $$VP-VT=(x+1)(\frac{2}{x+1}-1)+(y+1)(\frac{2}{y+1}-1)+\frac{z+1}{2}(\frac{2}{z+1}-2)$$

Ngoài ra có:$\frac{2}{x+1}-1;\frac{2}{y+1}-1>0$ và $x+1 \geq y+1 \geq z+1 >\frac{z+1}{2}$

Vậy thu được: $$VP-VT>\frac{z+1}{2}(\frac{2}{x+1}-1+\frac{2}{y+1}+\frac{2}{z+1}-2)=0$$

Đến bất đẳng thức thứ 2: Ta lấy vế phải trừ vế trái được:

$$VP-VT=x^n(1-x)+y^n(1-y)+z^n(1-z)>0$$

Từ 2 bất đẳng thức trên ta có: $$x^{n+1} + y^{n+1} + z^{n+1} <x^n+y^n+z^n < ...<x+y+z<2$$

Hay có điều phải chứng minh.

 

Ta hãy đến một số vẻ đẹp khác thông qua thêm lượng bên vế trái. Bạn đọc hãy thử chứng minh sau, khi nào có bài chứng minh đúng hay sai không quan trọng, mình sẽ đăng solution!! HIHI

 

Bài 1: $$\frac{a}{b+c}+\frac{b}{c+a}+\frac{c}{a+b}\geq \frac{3(a^2+b^2+c^2)}{(a+b+c)^2}+\frac{1}{2}.$$

Bài 2: $$\frac{a}{b+c}+\frac{b}{c+a}+\frac{c}{a+b}\geq \frac{3}{2}+\frac{a^2+b^2+c^2-ab-bc-ca}{a^2+b^2+c^2+ab+bc+Ca}$$

Bài 3: $$\frac{a}{b+c}+\frac{b}{c+a}+\frac{c}{a+b}+\frac{ab+bc+ca}{2(a^2+b^2+c^2)}\geq 2$$

Bài 4:  $$\frac{a}{b+c}+\frac{b}{c+a}+\frac{c}{a+b}\geq \frac{3(a+b+c)}{(\sqrt{a}+\sqrt{b}+\sqrt{c})^2}+\frac{1}{2}.$$

 

Rất cảm ơn ai đã đọc tới đây, mong mọi người sẽ ủng hộ bài giải nhiệt tình, và đóng góp thêm bài nhưng vui lòng đóng góp thêm sol ạ tại đóng góp bài không thì không biết nó có đúng không nữa hihi!!!




#741279 Tìm $lim \frac{n}{\sqrt[n]{n!}...

Gửi bởi truongphat266 trong 02-09-2023 - 09:14

Tìm $lim \frac{n}{\sqrt[n]{n!}}$

https://math.stackex...nn?noredirect=1




#741210 Chứng minh rằng $(abc)^2(a^2+b^2+c^2) \leq 3$

Gửi bởi truongphat266 trong 25-08-2023 - 21:01

Sửa đề lại nhé!

Cho $a,b,c>0$ và $a+b+c =3$. Chứng minh rằng $(abc)^2(a^2+b^2+c^2) \leq 3$

 

Có: $$(abc)^2(a^2+b^2+c^2)=(abc)^2[9-2(ab+bc+ca)] \leq (abc)^2[9-6(abc)^{\frac{2}{3}}]$$

Theo giả thiết thì có: $$abc \leq \frac{(a+b+c)^3}{27}=1$$

Đặt $(abc)^{\frac{2}{3}}=u$ $(u \in (0;1])$ thì ta cần chỉ ra: $$ u^3(9-6u) \leq 3 \Leftrightarrow (u-1)[3u(1-u)(2u+1)+3] \leq 0$$ 

Dẫn đến có điều phải chứng minh.

 

Tổng quát: Cho $a,b,c,k>0$ thỏa $a^k+b^k+c^k=3$

Tìm mọi $k$ sao cho bất đẳng thức sau đúng: $$(abc)^2[a^{k+1}+b^{k+1}+c^{k+1}] \leq 3$$




#741172 $\frac{1}{a+1} + \frac{1}{b...

Gửi bởi truongphat266 trong 23-08-2023 - 22:30

Cho a,b,c,d dương thoả a>=b>=c>=d và abcd=1. Tìm hằng số k nhỏ nhất để bđt sau đúng: 1/(a+1) + 1/(b+1) +1/(c+1) +k/(d+1) >= (3+k)/2

 

https://artofproblem...374684p19428567




#741157 KĨ THUẬT SỬ DỤNG BẤT ĐẲNG THỨC PHỤ

Gửi bởi truongphat266 trong 23-08-2023 - 13:31

 

I. MỘT SỐ BẤT ĐẲNG THỨC PHỤ THƯỜNG DÙNG

1. $\frac{a^2+b^2}{2}\geq \left(\frac{a+b}{2} \right)^2$, $\forall$$a,b\in \mathbb{R}$
2. $\frac{a^3+b^3}{2}\geq \left(\frac{a+b}{2} \right)^3$, $\forall$$a,b\geq 0$
3. $a^3+b^3\geq ab(a+b)$, $\forall$$a,b\geq 0$
4. $a^4+b^4\geq ab(a^2+b^2)$, $\forall$$a,b\geq 0$
5. $a^5+b^5\geq a^2b^2(a+b)$, $\forall$$a,b\geq 0$
6. $a^2+ab+b^2\geq \frac{3(a+b)^2}{4}$, $\forall$$a,b\in \mathbb{R}$
7. $\frac{a^2-ab+b^2}{a^2+ab+b^2}\geq \frac{1}{3}$, $\forall$$a,b\in \mathbb{R}$, $a^2+b^2\neq 0$
8. $(1+a)(1+b)\geq \left(1+\sqrt{ab} \right)^2$, $\forall$$a,b\geq 0$
9. $(1+a)(1+b)(1+c)\geq \left(1+\sqrt[3]{abc} \right)^3$, $\forall$$a,b,c\geq 0$
10. $\frac{1}{1+a^2}+\frac{1}{1+b^2}\geq \frac{2}{1+ab}$, với $ab\geq1$
II. BÀI TOÁN ỨNG DỤNG
Bài 1. Cho $a,b,c\in \mathbb{R^+}$ thỏa mãn $\frac{1}{a}+\frac{1}{b}+\frac{1}{c}\leq 3$. Tìm giá trị lớn nhất của biểu thức:
                        $P= \frac{1}{\sqrt{a^2-ab+3b^2+1}}+\frac{1}{\sqrt{b^2-bc+3c^2+1}}+\frac{1}{\sqrt{c^2-ca+3a^2+1}}$
Bài 2. Cho $a,b,c>0$ thỏa mãn $abc\leq 1$. Tìm giá trị nhỏ nhất của biểu thức:
                        $A=\frac{bc}{a^2b+a^2c}+\frac{ca}{b^2c+b^2a}+\frac{ab}{c^2a+c^2b}$
Bài 3. Cho $a,b,c\in \mathbb{R^+}$. Chứng minh rằng:
                       $\frac{a^2}{\sqrt{8a^2+3b^2+14ab}}+\frac{b^2}{\sqrt{8b^2+3c^2+14bc}}+\frac{c^2}{\sqrt{8c^2+3a^2+14ca}}\leq \frac{a+b+c}{5}$
Bài 4. Cho $a,b,c\in \mathbb{R^+}$. Chứng minh rằng:
                       $\frac{a^2+b^2}{a+b}+\frac{b^2+c^2}{b+c}+\frac{c^2+a^2}{c+a}\leq \frac{3(a^2+b^2+c^2)}{a+b+c}$
Bài 5. Cho $a,b,c>0$ thỏa mãn $ab+bc+ca=1$. Tìm giá trị nhỏ nhất của biểu thức:
                       $C=\frac{1}{4a^2-bc+2}+\frac{1}{4b^2-ca+2}+\frac{1}{4c^2-ab+2}$
Bài 6. Cho $a,b,c\in \mathbb{R^+}$ thỏa mãn $ab+bc+ca+abc=2$. Tìm giá trị lớn nhất của biểu thức:
                       $M=\frac{a+1}{a^2+2a+2}+\frac{b+1}{b^2+2b+2}+\frac{c+1}{c^2+2c+2}$
Bài 7. Cho $a,b,c\in \mathbb{R^+}$ thỏa mãn $a^2+b^2+c^2=3abc$. Chứng minh rằng:
                       $\frac{a^2}{b+2}+\frac{b^2}{c+2}+\frac{c^2}{a+2}\geq 1$
Bài 8. Cho $a,b,c\in \mathbb{R^+}$ thỏa mãn $ab+bc+ca$=5. Tìm giá trị lớn nhất của biểu thức:
                       $A=\frac{x}{\sqrt{x^2+5}}+\frac{y}{\sqrt{y^2+5}}+\frac{3z}{\sqrt{6(z^2+5)}}$
Bài 9. Cho $x,y,z\in \mathbb{R^+}$. Chứng minh rằng:
                       $\frac{\sqrt{xy}}{1+\sqrt{yz}}+\frac{1}{\sqrt{xy}+\sqrt{yz}}+\sqrt{\frac{2\sqrt{yz}}{1+\sqrt{xy}}}\geq 2$
Bài 10. Cho $a,b,\in \mathbb{R^+}$. Tìm giá trị lớn nhất của biểu thức:
                       $N=(a+b)\left(\frac{1}{\sqrt{a^2-ab+2b^2}}+\frac{1}{\sqrt{b^2-ab+2a^2}} \right)$

 

  Đếm hai cách thật khó  :(  :(  :(  :(  :(  :( sao tổ hợp lại khó vậy chứ
 

Bài 1.

Có: $$\sum \frac{1}{\sqrt{a^2-ab+3b^2+1}}=\sum \frac{1}{\sqrt{(a^2-ab+b^2)+b^2+(b^2+1)}} \leq \sum $\sum \frac{1}{\sqrt{b(a+b+2)}}=\sum \frac{2}{\sqrt{4b(a+b+2)}}\leq \sum (\frac{1}{4b}+\frac{1}{a+b+2})\leq \sum (\frac{1}{4b}+\frac{1}{16}(\frac{1}{a}+\frac{1}{b}+2))$$ 

Cộng lại được GTLN là: $$\frac{3}{2}\Leftrightarrow a=b=c=1$$

 

Bài 5.

 

Đặt: $$(ab;bc;ca) \mapsto (a;b;c)$$

 

Vậy: $$C=\sum \frac{1}{\frac{4ac}{b}-b+2}=\sum \frac{1}{\frac{4ac}{b}-b+2(a+b+c)}=\sum \frac{1}{\frac{4ac}{b}+2a+b+2c}=\sum \frac{b}{4ac+2ab+b^2+2bc}$$

 Nên: $C=\sum \frac{b}{(2a+b)(2c+b)} \geq \sum \frac{4b}{(2a+2b+2c)^2}=\sum \frac{b}{(a+b+c)^2}=1$

 

Bài 6.

 

Đặt: $$(x;y;z)=(\frac{a+1}{\sqrt{3}};\frac{b+1}{\sqrt{3}};\frac{c+1}{\sqrt{3}})$$

 

Có: $$M=\sum \frac{x}{3x^2+1}=\sum \frac{x}{3x^2+\frac{3xyz}{x+y+z}}=\sum \frac{x+y+z}{3(x+y)(x+z)}$$

 

Cần chứng minh: $$M\leq \frac{3}{4}\Leftrightarrow 9(x+y)(y+z)(z+x)\geq 8(x+y+z)^2$$

 

Áp dụng bất đẳng thức 8 phần 9 thì cần chứng minh: $$xy+yz+zx \geq x+y+z\Leftrightarrow xy+yz+zx\geq (x+y+z)\sqrt{\frac{3xyz}{x+y+z}}\Leftrightarrow \sum x^2(y-z)^2\geq 0$$

 

Bài 8. Chắc là lượng giác

 

Bài 9.

 

 Đặt: $$(\sqrt{xy};\sqrt{yz}) \mapsto (a;b)$$

 

Điều cần chứng minh tương đương:$$\frac{a}{1+b}+\frac{1}{a+b}+\sqrt{\frac{2b}{1+a}}\geq2$$

 

Mà: $$VT\geq \frac{a}{1+b}+\frac{1}{a+b}+\frac{4b}{1+a+2b}-2+2=\frac{(a-1)^2(a+1+b)}{(1+b)(a+b)(1+a+2b)}+2\geq 2$$




#741108 Giải phương trình vô tỉ sau: $\sqrt{x^{2}-3\sq...

Gửi bởi truongphat266 trong 19-08-2023 - 14:48

Giải phương trình vô tỉ sau:

$\sqrt{x^{2}-3\sqrt{2}x+9}+\sqrt{x^{2}-4\sqrt{2}x+16} = 5$

 

Đặt: $\sqrt{2}x=t$ phương trình đã cho viết lại:

$$\sqrt{(t-3)^2+3^2}+\sqrt{(4-t)^2+4^2}=5\sqrt{2}$$

 

Sử dụng bất đẳng thức Minkowski thì dấu bằng xảy ra, ra nghiệm.




#741059 $n=x!+y!$

Gửi bởi truongphat266 trong 15-08-2023 - 19:51

Có tồn tại một số tự nhiên $n$ có thể viết dưới dạng: $n=x!+y!$ với $x,y$ nguyên dương và $x \leq y$ bằng 2 cách khác nhau hay không?




#741017 Chứng minh $a^2 +4b^2<1$

Gửi bởi truongphat266 trong 12-08-2023 - 13:26

Cho $a,b$ là các số thực dương sao cho $a^3+b^3=a-b.$ CMR: $a^2 +4b^2<1$.

 

 

 

Có: $\frac{a^3+b^3}{a-b}=1$

Vậy tự nhiên nghĩ ngay cần chứng minh: $\frac{a^3+b^3}{a-b}> a^2+4b^2$

Nhưng điều này lại tương đương: $(a-2b)^2 +b^2 \ge 0$ (Đúng)

 

Đây là đề CGMO 2005!

 

Những bài mạnh hơn:

 

Gốc: Điều kiện như trên, chứng minh: $x^2 +y^2 <1$ (Dễ)

Chặt: Điều kiện như trên, chứng minh: $4a^2 - 5b^2 < 5$

Dấu bằng xảy ra: Điều kiện như trên, chứng minh: $a^2 + 2(\sqrt{2}+1)b^2 \leq 1$

 

Nâng bậc: Cho $a,b \geq 0$ và $a^5+b^5=a-b$. Chứng minh: $a^4+b^4<a^4+2b^4<1$

Điều kiện như trên, tìm $k$ lớn nhất sao cho bất đẳng thức sau đúng: $a^4 +kb^4 \leq 1$ (Tìm dấu bằng nữa)

Điều kiện như trên, chứng minh với $k=12$

Điều kiện như bài gốc, chứng minh: $(1+3ab)^3 \geq 27b^2$

Điều kiện như bài gốc, chứng minh: $(a+b)^2 \leq  (1-ab)(1+2ab+2a^2b^2)$

Điều kiện như bài gốc, chứng minh: $a^2 + 4b^2(1+b^2)^2 \leq 1$

 

__Những bài này chỉ cần biến đổi tương đương  :D




#740992 Tìm Max $M=xy+xz+xt+yz+yt+3zt$

Gửi bởi truongphat266 trong 10-08-2023 - 23:45

Cho các sổ thực x, y, z, t thỏa mãn $x^{2}+y^{2}+z^{2}+t^{2}=1$ . Tìm giả trị lớn nhất của biểu thức

$M=xy+xz+xt+yz+yt+3zt$

 

Có:$M=ab+(a+b)(c+d)+3cd\leq ab+\frac{\sqrt{5}+1}{4}(a+b)^2+\frac{\sqrt{5}-1}{4}(c+d)^2+3cd$

$\leq \frac{a^2+b^2}{2}+\frac{\sqrt{5}+1}{2}(a^2+b^2)+\frac{\sqrt{5}-1}{2}(c^2+d^2)+\frac{3}{2}(c^2+d^2)$

$=\frac{\sqrt{5}+2}{2}(a^2+b^2+c^2+d^2)=\frac{\sqrt{5}+2}{2}$

Vậy $Max_M=\frac{\sqrt{5}+2}{2}$ khi $(a^2;b^2;c^2;d^2)=(\frac{5-\sqrt{5}}{20};\frac{5-\sqrt{5}}{20};\frac{5+\sqrt{5}}{20};\frac{5+\sqrt{5}}{20})$

 

Bất đẳng thức sau khó hơn: Cho $a,b,c,d$ là các số thực. Tìm giá trị nhỏ nhất của:

$$P=x^2+y^2+z^2+t^2+xy+xz+xy+yz+yt+zt+x+y+z+t$$

 

Đáp án: $\frac{-2}{5}$

 

Đơn giản hơn: Điều kiện như trên mà đổi thành 3 biến. (Đáp án: $\frac{-3}{8}$)

 

Dọa người khác: Điều kiện như trên mà đổi thành 5 biến. (Đáp án: $\frac{-5}{12}$)

 

Chặt hơn: Cho $a,b,c,d$ dương. Tìm giá trị nhỏ nhất của:

$$T=\frac{(a+b+c+d)^2+1}{2ab+ac+ad+bc+bd}$$

 

Đáp án: $\sqrt{5}-1$

 

Tổng quát: Cho $a_1,a_2,..,a_n$ dương. Chứng minh rằng:

 

$$\sum_{i=1}^{n}(a_i+a_i^2)+\sum_{1\leq p \leq q \leq n}^{a_pa_q}\geq -\frac{n}{2(n+1)}$$

 

Hint: Chứng minh bằng Bunhia! Dấu bằng xảy ra khi: $a_i=- \frac{1}{n+1}$

 

Mạnh hơn: Điều kiện như bài gốc, chứng minh các bất đẳng thức sau:

$$ab+ac+ad+bc+bd+cd \leq 1+8abcd$$

$$ab+ac+ad+bc+bd+cd \leq \frac{3+ 3 \sqrt{3}}{8}+ (18-6 \sqrt{3})abcd$$.




#740991 Tìm $x,y$ là 2 số nguyên dương biết $x^3-y^3 = xy + 61$

Gửi bởi truongphat266 trong 10-08-2023 - 23:28

tìm $x,y$ là 2 số nguyên dương thỏa mãn:

$x^3-y^3 = xy + 61

Một đề thi năm nào đó của nước Nga.

Do vai trò $x,y$ đặt: $x=y+k$ trong đó $k$ nguyên dương.

Phương trình đã cho tương đương:

$$(3k-1)y^2 + k(3k-1)y + (k^3-61)=0$$

Do $VP=0$ mà 2 nhân tử trước đó lớn hơn $0$ nên: $k^3-61<0$

Điều này dẫn tới $k \in [1;3]$

Thế từng trường hợp vào giải được: $(x,y)=(6,5)$




#740875 $a^4b +b^4c + c^4d +d^4a \ge abcd(a+b+c+d)$

Gửi bởi truongphat266 trong 02-08-2023 - 21:36

Cho $a,b,c,d \ge 0$. Chứng minh rằng: $$a^4b +b^4c + c^4d +d^4a \ge abcd(a+b+c+d)$$

 




#739706 Ứng dụng hình học phẳng THPT cho THCS

Gửi bởi truongphat266 trong 30-05-2023 - 19:11

Cuối cùng cũng sắp thi xong cấp 3 rồi nên hôm nay em xin được phép đăng lên diễn đàn và góp vui với mọi người cũng như dành cho các bạn sắp vào lớp 9 sắp tới về hàng điểm điều hòa trong hình học THPT có thể đem về để giải THCS vô cùng thú vị.
Nếu có sai sót, mong mọi người sẽ giúp đỡ. Và mong mọi người sẽ ủng hộ và đóng góp nhiệt tình cho bài đầu của em  :D

Ngoài ra, mong mọi lời giải trong đây sẽ đều là thuộc về THCS.
Chủ đề 1. Hàng điểm điều hòa
~~~Định nghĩa~~~
Cho 1 đoạn thẳng $d$ có 4 điểm $A,C,B,D$ thỏa mãn tỉ số: $\frac{CA}{CB}=\frac{DA}{DB}$ 

$\rightarrow A,B,C,D$ gọi là hàng điểm điều hòa.

Một số kí hiệu thường dùng:

$(ABCD)$: hàng điểm $A,B,C,D$

$O(ABCD)$: Chùm 4 đoạn thẳng cắt nhau tại $O$

Đây là một số hệ quả rút ra từ điều trên mà hay gặp ở câu b) hình ở đề lớp 9 đại trà:

Cho $A,B,C,D$ là hàng điểm điều hòa. Gọi $M$ là trung điểm $AB$, $J$ là trung điểm $CD$.

Chứng minh rằng:

i) $MB^2=MA^2=MC.MD$ 

ii) $AC.AD=AB.AN$ 

iii) $\frac{1}{AC}+\frac{1}{AD}=\frac{2}{AB}$ 

Ứng dụng:

Từ điểm $P$ nằm ngoài $(O)$. Kẻ các tiếp tuyến $SA,SB$ tới $(O)$ ($A,B$ là các tiếp điểm). Kẻ cát tuyến $SMN$. Qua $M$ kẻ đường thẳng song song với $SA$ cắt $AB,AN$ tại $X,Y$. Chứng minh $XM=XY$.